Добавил:
Опубликованный материал нарушает ваши авторские права? Сообщите нам.
Вуз: Предмет: Файл:
Скачиваний:
232
Добавлен:
06.05.2013
Размер:
1.01 Mб
Скачать
  1. Теорема Больцано-Вейрштрасса.

Теорема 6.2.(Больцано-Вейерштрасса). Из любой ограниченной последовательности можно выделить сходящуюся подпоследовательность.

Доказательство: Пусть {xn} - ограниченная последовательность, то есть все ее элементы лежат на некотором сегменте [a ,b].

a xnb(n).

(здесь рисунок)

Разделим сегмент [a ,b] пополам. По крайней мере на одной из половин сегмента [a ,b] лежит бесконечно много членов последовательности {xn}, обозначим эту половину через [a1 ,b1]. Возьмем какой-нибудь :a1 b1. Далее разделим сегмент [a1 ,b1] пополам и обозначим через [a2 ,b2] ту половину, на которой находится бесконечно много членов последовательности {xn}. Выберем[a2 ,b2],k2>k1.a2 b2. Затем разделим сегмент [a2 ,b2] пополам, и так далее. Продолжая этот процесс, получим стягивающуюся систему сегментов [a1 ,b1], [a2 ,b2], … , [an,bn], … (так какbn-an= 0 приn), и последовательность , которая является подпоследовательностью последовательности {xn}.

n: an bn. (1)

По теореме 6.1 (Теорема 6.1. Существует, и притом только одна, точка, принадлежащая всем сегментам стягивающейся системы)точкас:liman=limbn=c. Отсюда и из неравенства (1) следует, что cприn . Таким образом, мы выделили из последовательности {xn} сходящуюся подпоследовательность.

Теорема доказана.

Определение. Последовательность {xn} называется неограниченной, еслиA > 0n:xn>A.

//Замечание: Для неограниченных последовательностей теорема Больцано-Вейерштрасса неверна.

Примеры.

  1. {n} = 1, 2, 3, …,n, …

Из {n} нельзя выделить сходящуюся подпоследовательность.

  1. {xn} = 1, 0, 2, 0, 3, 0, … ,n, 0, …

{xn} - неограниченная подпоследовательность.

{x2n}0.

  1. Критерий Коши сходимости последовательности.

Определение:Последовательность {xn} называется фундаментальной, если> 0N,n>Nинатуральногоp:xn+p-xn<.

Так как m=n+p- тоже произвольное число >N, то определение фундаментальности можно сформулировать следующим образом:> 0N,n>Nиm>N:xm-xn<.

Геометрически фундаментальность последовательности {xn} означает, что для любого сколь угодно малогосуществует такой номерN, что любые два члена последовательности с большим, чемN, номерами, отстоят друг от друга не более, чем на.

(здесь рисунок)

Пример: Докажем, что последовательность =1,,, … - фундаментальная. Зададим произвольное> 0, возьмемN> . Тогда>, иn>Nинатуральногоp:xn+p-xn= =-< <<. Это и означает, что последовательность- фундаментальная.

Лемма 2. Фундаментальная последовательность ограничена.

Доказательство: Пусть {xn} - фундаментальная последовательность. Возьмем какое-нибудь положительное, например,= 1. По определению фундаментальности,N,n>Nиm>N:xm-xn< 1. Зафиксируем какое-нибудьm0>N, тогда< 1 приn>N, или - 1 <xn< + 1 приn>N. Таким образом, все члены последовательности с номерамиn>Nлежат в интервале (- 1, + 1), вне этого интервала лежит только конечное число членов последовательности. Это и означает, что последовательность {xn} ограничена.

Лемма доказана.

Теорема 6.4(критерий Коши сходимости последовательности) Для того, чтобы числовая последовательность сходилась, необходимо и достаточно, чтобы она была фундаментальной.

Доказательство.

  1. Необходимость. Дано: Последовательность {xn} сходится. Требуется доказать, что последовательность {xn} - фундаментальная. Пусть =a. Зададим произв.> 0.

По определению предела, N,n>N:xn-a< , иm>N:xm-a< . Еслиn>N,m>N, тоxm-xn=(xm-a) - (xn-a) +<.

Это и означает по определению, что последовательность {xn} - фундаментальная.

Необходимость доказана.

  1. Достаточность. Дано: Последовательность {xn} - фундаментальная. Требуется доказать: {xn} сходится. По лемме 2, последовательность {xn} ограничена, следовательно, можно выделить сходящуюся подпоследовательность . Пусть =a. Докажем, что =a. Зададим произвольное> 0. Так как подпоследовательность сходится кa, начиная с некоторого номераN1все члены{- окрестности точкиa}, а так как последовательность {xn} - фундаментальная, то начиная с некоторого номераN2все членыxnотстоят от членовменьше, чем на . Следовательно, начиная с номераN=max(N1, N2) все члены последовательностиxn{- окрестности точкиa}, а это и означает, что =a, что и требовалось доказать.

Теорема доказана.

Пример: Рассмотрим последовательность {sinn}. Докажем, что эта последовательность расходится. Для этого достаточно доказать, что она не является фундаментальной. Предположим противное: допустим, что {sinn} - фундаментальная. Тогда если> 0N,n>Nинатуральногоp:xn+p-xn=sin(n+p) -sinn<. Возьмемp= 2.

sin(n+2) - sin n< 2sin 1cos(n+1)< .

  > 0  N,  n >N: cos(n+1)< .

=> {cosn} - бесконечно малая, то естьcosn0 приn.

cos(n+1) = cos ncos 1- sin nsin 1.

sin n = ( cosncos 1 - cos(n+1))  0 при n  .

cosnиsinn0 приn. Но это противоречит тому, чтоcos2n+sin2n=1.

Полученное противоречие доказывает, что последовательность {sinn} расходится.